[tex] \frac{15x - 8 }{3} + \frac{9x - 3}{2} [/tex]

Answers

Answer 1

Greetings from Brasil...

Here is the problem:

[(15X - 8)/3] + [(9X - 3)/2]

Let's multiply everything by 6

[6.(15X - 8)/3] + [6.(9X - 3)/2]

[2.(15X - 8)] + [3.(9X - 3)]

30X - 16 + 27X - 9

57X - 25

Related Questions

Ernesto solves the equation below by first squaring both sides of the equation. \sqrt{\dfrac{1}{2}w+8}=-2 2 1 ​ w+8 ​ =−2square root of, start fraction, 1, divided by, 2, end fraction, w, plus, 8, end square root, equals, minus, 2 What extraneous solution does Ernesto obtain?

Answers

Answer:

w = -8

Step-by-step explanation:

Given the equation solved by Ernesto expressed as [tex]\sqrt{\dfrac{1}{2}w+8}=-2[/tex], the extraneous solution obtained by Ernesto is shown below;

[tex]\sqrt{\dfrac{1}{2}w+8}=-2\\\\square\ both \ sides \ of \ the \ equation\\(\sqrt{\dfrac{1}{2}w+8})^2=(-2)^2\\\\\dfrac{1}{2}w+8 = 4\\\\Subtract \ 8 \ from \ both \ sides\\\\\dfrac{1}{2}w+8 - 8= 4- 8\\\\\dfrac{1}{2}w= -4\\\\multiply \ both \ sides \ by \ 2\\\\\dfrac{1}{2}w*2= -4*2\\\\w = -8[/tex]

Hence, the extraneous solution that Ernesto obtained is w = -8

The value of which of these expressions is closest to e?
33
31
B.
O A. (1-5
+39)
(1+0)
(1+3)
32
C.
134
D.

Answers

Answer:

(1 +1/34) to the power of 34

Step-by-step explanation:

help me please !! find the domain and range of the function represented by the graph. determine wether the domain is discrete or continuous.

Answers

Answer:

Domain is the whole R and the range is R+. The domain is continuous

The local pizza restaurant offers free delivery if you live within a 6-mile radius of the restaurant. The restaurant is located at the origin. Ada’s house is 4 miles west and 5 miles north of the restaurant at point (–4, 5). Does Ada’s house lie on or within the circle representing the area that gets free delivery?

No, the distance from the restaurant to Ada’s house is miles, which is greater than the 6-mile maximum radius.
Yes, the distance from the restaurant to Ada’s house is exactly 6 miles.
No, the distance from the restaurant to Ada’s house is 7 miles, which is greater than the 6-mile maximum radius.
Yes, the distance from the restaurant to Ada’s house is miles, which is less than the needed 6-mile radius.

Answers

Answer:

No adda doesnt get free delivery

Step-by-step explanation:

Answer: A) No, the distance from the restaurant to Ada’s house is Square Root of 41 miles, which is greater than the 6-mile maximum radius.

Step-by-step explanation:

As a burglar reaches for something on the mantel, accidentally knocks over a clock. It falls to the floor, breaks, and stops. The next morning, however, pol.ice aren't able to determine what time the robbery took place. Why not?

Answers

Answer: There is no way of finding out if the clock worked before then, and if it stopped at P.m or A.m.

Step-by-step explanation:

Let's say the robber broke the clock at 3 a.m, the police would first of all not have any idea if the clock was indeed broken before the robber knocked it over, and the clock has no way of saying whether it was broken at 3 a.m. or 3 p.m, therefore it's rendered useless.

What is the value of x in the equation 3x-4y=65, when y=4?
x=13 1/4
x=21 2/3
x =23
x = 27

Answers

Hello there! :)

Answer:

[tex]\huge\boxed{x = 27}[/tex]

Given the equation:

3x - 4y = 65 where y = 4

Substitute in 4 for "y":

3x - 4(4) = 65

Simplify:

3x - 16 = 65

Add 16 to both sides:

3x - 16 + 16 = 65 + 16

3x = 81

Divide both sides by 3:

3x/3 = 81/3

x = 27.

Answer: x = 27

Explanation:

3x - 4y = 65
3x - 4(4) = 65
3x - 16 = 65
3x = 65 + 16
3x = 81
x = 81/3
x = 27

What is the answer to this question +8+(+4)

Answers

Answer:

12 is the answer of this questions.

Step-by-step explanation:

Thank you for you question.

In a circle, an arc measuring 130° is what percentage of the circumference of the circle

Answers

Answer:

≈ 36.1%

Step-by-step explanation:

In any circle the following ratio is equal

[tex]\frac{arc}{circmference}[/tex] = [tex]\frac{centralangle}{360}[/tex] = [tex]\frac{130}{360}[/tex] , thus

percentage = [tex]\frac{130}{360}[/tex] × 100% ≈ 36.1%

an arc measuring 130° is approximately 36.11% of the circumference of the circle.

To find the percentage of the circumference that an arc measuring 130° represents, we need to calculate the ratio of the arc length to the circumference of the circle and then convert it to a percentage.

The circumference of a circle is given by the formula C = 2πr, where r is the radius of the circle.

Let's assume the radius of the circle is r.

The circumference of the circle is C = 2πr.

To find the length of the arc corresponding to 130°, we need to calculate the fraction of the total angle (360°) that 130° represents:

Fraction of the angle = (130° / 360°) = (13/36).

Since the fraction of the angle is equal to the fraction of the arc length to the circumference, the length of the arc can be calculated as:

Arc length = Fraction of the angle * Circumference = (13/36) * (2πr).

Now, to find the percentage of the circumference that the arc length represents, we divide the arc length by the circumference and multiply by 100:

Percentage = (Arc length / Circumference) * 100

Percentage = [(13/36) * (2πr)] / (2πr) * 100

Percentage = (13/36) * 100

Percentage = 36.11%

Therefore, an arc measuring 130° is approximately 36.11% of the circumference of the circle.

Learn more about arc length here

https://brainly.com/question/10266974

#SPJ2

Plz ans ASAP! Tysm! Pzlllzzz!

Answers

Answer:

Step-by-step explanation:

Number of class intervals = 7

Class width = 5

Class intervals (heights)        Frequency (Number of boys)

130 - 135                                                     3                        

136 - 140                                                     5

141 - 145                                                      3

146 - 150                                                     5

151 - 155                                                      2

156 - 160                                                     1

161 - 165                                                      1

Therefore, frequency table given above will represent the distribution of the heights and number of students.

Bill spent $42 on fruit at the grocery store. He spent a total of $60 at the store. What percentage of the total did he
spend on fruit?​

Answers

Answer:

70%

Step-by-step explanation:

To find the total percentage of his $60 dollars that he spent on fruit, we simply take the amount of money spent on fruit divided by the total spent.

% spent on fruit = 42 / 60

% spent on fruit = 0.7

% spent on fruit = 70 %

Cheers.

Answer:

70%

Step-by-step explanation:

so he spent 42/60

lets simplify by dividing on both sides by 2

21/30

then divide by 3

7/10

which is also 70/100

so 70%

yaaaaayyyy we r done!!!!!!!!!

HOPE I HELPED

PLS MARK BRAINLIEST

DESPERATELY TRYING TO LEVEL UP

  ✌ -ZYLYNN JADE ARDENNE

JUST A RANDOM GIRL WANTING TO HELP PEOPLE!

                                 PEACE!

Which function has a range of y < 3? y=3(2)x y = 3(3)x y = -(2)x+ 3 y = (2)x-3

Answers

Answer:

Third option is the correct answer

[tex]y = -(2)x+ 3[/tex]

Step-by-step explanation:

Given functions are:

[tex]y=3(2)x\\ y = 3(3)x\\ y = -(2)x+ 3\\ y = (2)x-3[/tex]

To find:

The function which has a range [tex]y <3[/tex] ?

Solution:

First of all, let us learn about the terms Domain and Range for a function:

[tex]y = f(x)[/tex]

Domain is the value of [tex]x[/tex] which is given as input to the function and gives a valid value of output as [tex]y[/tex] (i.e. function is defined).

Range of a function is the value [tex]y[/tex] that comes as output when given a valid value of [tex]x[/tex] as input.

Now, let us consider the given options above.

Third option is the correct answer.

[tex]y = -(2)x+ 3[/tex]

As we can see, [tex](2)x[/tex] is subtracted from 3 so [tex]y[/tex] will have a value which is lesser than 3.

i.e. range will be [tex]y <3[/tex].

No other function has such condition present in it.

[tex]y = -(2)x+ 3[/tex] is the correct answer.

idk know the answer i really need help

Answers

Answer:

37x - 13

Step-by-step explanation:

Hello!

The product of 37 and x mean multiply so we can write it as

37x

13 less means subtract so we put that at the end

37x - 13

The answer is 37x - 13

Hope this helps!

Answer:

37 * x - 13

This answer is here according to your question It said that 37*x - 13 Please mark this answer The brainliest one

What is 10+x-15-3x please

Answers

Answer:

-2x-5

Step-by-step explanation:

10+x-15-3x

Combine like terms

10-15   +x-3x

-5 -2x

We normally put the variable first

-2x-5

Calculate the sum and enter it below.
8+ (-12)

Answers

Answer:

-4 is the answer

Answer:

-4

Step-by-step explanation:

8+(-12)

+ x - = -

8-12

-4

Calculate:
a) QR
b) PS
c) The area of quadrilateral PQRS

Answers

Greetings from Brasil...

For QR: we need to use the Sine Law in Any Triangle....

QS/SEN 72 = QR/SEN 25

6/SEN 72 = QR/SEN 25

6,3 = QR/SEN 25

QR = 2,66

For PS: we need to use the Cosine Law in Any Triangle....

PS² = PQ² + QS² - 2.PQ.QS.COS Q

PS² = 7,4² + 6² - 2.(7,4).6.COS 34

PS² = 90,76 - 73,61

PS = √17,14

PS = 4,14

For area we use Heron's Formula 2x.....

for ΔQRS = A1:

A1 = √[P.(P - QR).(P - RS).(P - QS)]

where P = (QR + RS + QS)/2

A1 = √[P.(P - QR).(P - RS).(P - QS)]

     RS = 6,26 (using RS/SEN 97 = QS/SEN 72)

P = (2,66 + 6,26 + 6)/2

P = 14,92/2 ⇒ P = 7,46

A1 = √[7,46.(7,46 - 2,66).(7,46 - 6,26).(7,46 - 6)]

A1 = 7,92

for ΔPQS = A2:

A2 = √[P.(P - PQ).(P - PS).(P - QS)]

P = (7,4 + 4,14 + 6)/2 = 8,77

A2 = √[8,77.(8,77 - 7,4).(8,77 - 4,14).(8,77 - 6)]

A2 = 12,41

Total Area = A1 + A2

Total Area = 7,92 + 12,41

Total Area = 20,33

see more:

https://brainly.com/question/17138076

— 12 — бр – (-2)
HELP ASAP

Answers

Answer:

-10 - 6p

Step-by-step explanation:

-12 - 6р - (-2)

~Combine like terms

-10 - 6p

Best of Luck!

[tex]\huge\text{Hey there!}[/tex]

[tex]\huge\textsf{-12 - 6p - (-2)}\\\\\huge\text{COMBINE the LIKE TERMS}\\\huge\textsf{\underline{\underline{-12 - (-2)}} + \underline{\underline{\underline{(-6p)}}}}\\\\\\\huge\text{\underline{\underline{-12 - (-2)}}}\\\huge\textsf{= -12 + 2}\\\huge\textsf{= \bf -10}\\\\\\\huge\textsf{-6p \text{does not have any like terms so it}}\\\huge\textsf{stays the same}\\\\\\\huge\textsf{= \bf -6p - 10}\\\\\boxed{\boxed{\huge\text{Answer: \bf -6p - 10}}}\huge\checkmark[/tex]

[tex]\huge\text{Good luck on your assignment \& enjoy your day!}[/tex]

~[tex]\boxed{\huge\boxed{\frak{Amphitrite1040:)}}}[/tex]

Can anyone help me with this ??

Answers

Answer:

y=x+-5

Step-by-step explanation:

y is x+-5 because as you can see in the graph the difference in everysingle one is -5 so y=x-5

Answer:

y=x+(-5)

Step-by-step explanation:

d=-3-(-4)=-3+4=1

y-(-4)=1(x-1)

y+4=x-1

y=x-1-4

or y=x+(-5)

(x - y) + 2y + x3, when x = -3 and y=7
plss help​

Answers

-5 is the answer (or at least I think its is..) since x = -3 and y = 7, thus it will become (-3 - 7) + 2(7) + 3(-3)
That is really hard

What is the value of the expression below when z=4?
10z-3

Answers

Answer:

37

Step-by-step explanation:

10z - 3 = 10(4) - 3

=40-3

=37

Answer:

37

Step-by-step explanation:

10*4-3 (z=4)

40-3

37

Hope it helps you

The formula for the area (A) of a circle is A = π • r2, where r is the radius of the circle. Ronisha wants to find the area of a sector of a circle that has a central angle of π6 radians. Enter the number that Ronisha should multiply by to find the area of the sector of the circle. Round your answer to the nearest hundredth.

Answers

Greetings from Brasil...

Here we will apply rule of 3...

This formula, A = πR²,  its for the whole circle, that is, 360° (2π)

We want the area of only a part, that is, a circular sector whose angle π/6

 sector           area            

    2π     -----    πR²

    π/6    -----     X

2πX = (π/6).πR²

2πX = π²R²/6

X = π²R²/12π

X = πR²/12

If Ronisha multiply the area by 1/12 she will get the area of sector with angle of π/6

Katherine's class is selling raffle tickets for $3 to raise money for charity. Katherine's class raised $504. Which equation would you use to find the number of tickets sold?


Answers

Answer:

3T=504

Step-by-step explanation:

HOPE I HELPED

PLS MARK BRAINLIEST

DESPERATELY TRYING TO LEVEL UP

        ✌ -ZYLYNN JADE ARDENNE

JUST A RANDOM GIRL WANTING TO HELP PEOPLE!

                                       PEACE!

Ayuda con esto de geometría

Answers

Answer:

Baba

Step-by-step explanation:

Booey

Joe travels at a constant speed of 80 miles per hour for ten hours. How many miles will he travel in ten
hours? How many kilometers will he travel in ten hours? (1 mi= 1.6 km)

Answers

Answer:

1287.48 km

Step-by-step explanation:

80 miles = 128.748 km

Distance = Speed x Time

Putting in 'x' for distance

x = 80*10  

x = 800 miles

or

x = 1287.48 km

After years of maintaining a steady population of 32,000, the population of a town begins to grow exponentially. After 1 year and an increase of 8% per year, the population is 34,560. Which equation can be used to predict, y, the number of people living in the town after x years? (Round population values to the nearest whole number.) y = 32,000(1.08)x y = 32,000(0.08)x y = 34,560(1.08)x y = 34,560(0.08)x

Answers

Answer:

y = 32,000(1.08)^x

Step-by-step explanation:

The exponential growth equation is y = a(1 + r)^x, where a is the initial amount, r is rate as a decimal, and x is the time.

In this situation, 32,000 is the initial amount (a) and 0.08 is the rate (r)

If we plug these into the equation, we get the equation y = 32,000(1.08)^x

So, y = 32,000(1.08)^x is the correct answer.

Answer:

A

Step-by-step explanation:

on edge 2020

A runner jogged 4 miles in ten laps around a circular
track. What is the greatest distance straight across
the track?

Answers

Answer:

.1273 mi = 627.2 feet

c = 2[tex]\pi[/tex]r

.4 = 2[tex]\pi[/tex]r

r = [tex]\frac{.4}{2\pi }[/tex]

r = [tex]\frac{.2}{\pi }[/tex]

d = [tex]\frac{.4}{\pi }[/tex]

d = .1273 mi = 627.2 feet

Step-by-step explanation:

What’s the function of the Unit Circle and why is it called the unit Circle?

Answers

Answer:

It is a unit of radius that is radius of 1. Thus, the distant to the middle to any edge is always 1.

Step-by-step explanation:

Musah stands at the center of a rectangular field.He takes 50 steps north,then 25 steps West and finally 50 on a bearing of 315°. Sketch Musah's movement How far west is Musah's final point from the center? How far north is Musah's final point from the center?

Answers

Answer:

The distance of  Musah's final point from the center in the west direction is   60.355 steps

The distance of  Musah's final point from the center in the north direction is   85.355 steps

Step-by-step explanation:

Given that :

Musah stands at the center of a rectangular field.

He takes 50 steps north, then 25 steps West and finally 50 on a bearing of 315°.

The sketch for Musah's movement is seen in the attached file below.

How far west is Musah's final point from the centre?

In order to determine how far west  is Musah's,

Let d be the distance of how far west;

Then d = BC + CD cos θ

In the North West direction,

cos θ = cos 45°

d = 25 + 50( cos 45°)

d = 25 + 50([tex]\dfrac{1}{\sqrt{2}}[/tex]  )

d = 25 + 50( 0.7071)

d =25 + 35.355

d = 60.355 steps

How far north is Musah's final point from the center?

Let d₁ be the distance of how far North;

Then d₁ = AB + CD sin  θ

d₁ = 50 + 50  sin  45°

d₁ = 50 + 50([tex]\dfrac{1}{\sqrt{2}}[/tex]  )

d₁ = 50 + 50( 0.7071)

d₁ = 50 + 35.355

d₁ = 85.355  steps

Please someone help ASAP!!!!!!

Answers

Answer:

1.7 million in Northern Ireland

Step-by-step explanation:

Simply do the difference between the number of people in the UK, minus the number of people in everywhere else except northern Ireland. That is:

60.2 M - 50.4 M - 5.1 M - 3.0 M = 1.7 M

Answer:

[tex]\boxed{1.7 million}[/tex]

Step-by-step explanation:

Hey there!

To find the amount of people who lived in Northern Ireland in 2005 we need to subtract the total 60.2 mil by the England, Scotland, and Wales people.

50.4 + 5.1 + 3

= 58.5

Now we can set up the following equation,

NI = 60.2 - 58.5

NI = 1.7 million

Hope this helps :)

Use a calculator to find the mean of the data. {21.6, 25, 20.6, 24.6, 20.3, 25.2, 22.8, 24.1, 22.5, 22.5, 22.6, 22, 24.6, 23.6, 23.5, 22, 24.4, 20.9, 21.4, 20.5, 23.2}

Answers

Answer:

22.8

Step-by-step explanation:

I just did the math :)

The mean of a given set of data is 22.8.

What is the arithmetic mean?

Arithmetic mean is defined as the ratio of the sum of observations to the total number of observations. It can be referred to as the average of a specific set of data or the arithmetic mean. One of the basic methods used to acquire a result, the term "Mean" is commonly used in the field of statistics.

The formula for the mean of a given set of data is as follows:

Mean = Sum of Observations/Total number of observations

Given the set of observations as :

{21.6, 25, 20.6, 24.6, 20.3, 25.2, 22.8, 24.1, 22.5, 22.5, 22.6, 22, 24.6, 23.6, 23.5, 22, 24.4, 20.9, 21.4, 20.5, 23.2}

Here, n = 21

⇒ ∑x = Sum of Observations

⇒ ∑x = 478.8

Mean = ∑x/n

Mean = 478.8/21

Mean = 22.8

Hence, the mean of the given data is 22.8.

Learn more about what Arithmetic means here:

https://brainly.com/question/13000783

#SPJ5

Solve the equation
(If possible please show work)

Answers

Answer:

Step-by-step explanation:

-8 + 8a = -12 + 4a

4a - 8 = -12

4a = -4

a = -1

Answer:

a = -1

Step-by-step explanation:

[tex]4(-2+2a) = -12 +4a\\-8+8a=-12+4a\\8a = -4 +4a\\4a=-4\\\frac{4a}{4} =\frac{-4}{4} \\a=-1[/tex]

I hope that makes since... I tried to show my work the best I could.

Other Questions
Show that the equations x^2-7x+6=0 and y^2-14y+40=0 form a rectangle.Also find the joint equations of diagonals. respect the teacher into passive In two or more complete sentences, explain how to use ordered pairs of points in f(x) = 3x - 5 and g(x) = x+5/3 to determine if f(x) and g(x) are inverses of each other Your view about discrimination.if you experience discrimination, how would you react.are you going to fight or you just going to ignore it? Explain in 5 sentences PAKI ANSWER PO ipapass ko po yan bukas nagsisimula na po ang skela namin The stock in Bowie Enterprises has a beta of .87. The expected return on the market is 11.70 percent and the risk-free rate is 2.89 percent. What is the required return on the company's stock The British Agricultural Revolution resulted in many farmers losing their jobs. Which sentence best explains the change that farmers experienced? If the ratio of two supplementary angles is 10 to 47, calculate the measure of the larger of the two angles. the box plots shows the price for two different brands of shoes An unpolarized beam of light with an intensity of 4000 W/m2 is incident on two ideal polarizing sheets. If the angle between the two polarizers is 0.429 rad, what is the emerging light intensity Suppose that we have the following information concerning the government's finances and the macroeconomy for a given year: Government Debt: $12 trillion Inflation: 10% Nominal Deficit: $1.5 trillion What is the real deficit for the year The difference of two trinomials is x2 10x + 2. If one of the trinomials is 3x2 11x 4, then which expression could be the other trinomial? 2x2 x 2 2x2 + x + 6 4x2 + 21x + 6 4x2 21x 2 Farmers and ranchers are considered to be part of the ________ which is the subdivision of the food industry that produces agricultural commodities. A piece of solid metal is put into an aqueous solution of . Write the net ionic equation for any single-replacement redox reaction. Assume that the oxidation state of in the resulting solution is 2 . Explain the importance of international employment Complete parts (a) through (c) below. (a) Determine the critical value(s) for a right-tailed test of a population mean at the alpha = 0.10 level of significance with 15 degrees of freedom. (b) Determine the critical value(s) for a left-tailed test of a population mean at the alpha = 0.01 level of significance based on a sample size of n = 20. (c) Determine the critical value(s) for a two-tailed test of a population mean at the alpha = 0.05 level of significance based on a sample size of n = 14. What is the US TREASURY BONDS pretax expected return? Suppose you roll a fair six-sided die 25 times. What is the probability that you roll 5 or more 6s on that die? A. 0.3883 B. 0.5937 C. 0.5 D. 0.4063 In January 2022, the management of Blossom Company concludes that it has sufficient cash to purchase some short-term investments in debt and stock securities. During the year, the following transactions occurred. Jan. 1 Purchased 75 $1,000, 8% TRC bonds for $75,000. Interest is payable annually on December 31. Feb. 1 Purchased 1,295 shares of LAF common stock for $53,095. Mar. 1 Purchased 540 shares of NCL common stock for $18,900. July 1 Received a cash dividend of $0.80 per share on the LAF common stock. Aug. 1 Sold 217 shares of LAF common stock at $40 per share. Sept. 1 Received $2 per share cash dividend on the NCL common stock. Dec. 31 Received the annual interest on the TRC bonds. 31 Sold the TRC bonds for $77,665. At December 31, the fair values of the LAF and NCL common stocks were $37 and $28 per share, respectively. These stock investments by Blossom Company provide less than a 20% ownership interest.Required:Journaline the transactions and post to the accounts Debt Investments and Stacklestments. A 6-month-old infant is unresponsive. You begin checking for breathing at the same time you check for the infant's pulse. Which is the maximum time you should spend when trying to simultaneously check for breathing and palpate the infant's pulse before starting CPR Instructions: Complete the verb by dragging each ending to its correct place. ( Look at the picture) Will Mark Brainliest. Only answer if you know how to do this assignment please.